PDA

Archiv verlassen und diese Seite im Standarddesign anzeigen : Lorentzkraft


George_05
11.06.11, 05:11
Hallo,

meine Frage bezieht sich auf bewegte Ladungsträger im Magnetfeld. Ich weiss, dass
die Lorentzkraft auf die Teilchen als resultierende Kraft wirkt und sie
auf eine Kreisbahn ablenkt und so als Zentripetalkraft wirkt.
Nun muss es nach Newton eigentlich eine Gegenkraft zu der Lorentzkraft geben, welche
vom Kreismittelpunkt aus auf die Teilchen wirkt und vom Betrag gleich groß ist.

Meine Frage lautet; welche Kraft wirkt in diesem Fall als Gegenkraft?

Vielen Dank


George

richy
11.06.11, 13:26
Hi
Ich meine nicht dass es eine Gegenkraft geben muss. Bei einer Kreisbewegung z.B. eine Person in einem Karusell, die man von "ausserhalb" des Karusells beobachtet, existiert auch keine Gegenkraft, Zentrifugalkraft. Lediglich wenn man sich in das Karusell begibt ergibt sich eine Zentrifugalkraft aus der Koordinatentransformation und fuehrt dazu, dass vom Beobachtersystem "Karusell" aus betrachtet die Person sich in Ruhe befindet.
F_zentri=F_petal, Summe F=0, keine Bewegung

Von ausserhalb betrachtet :
F_petal=m*a und a ist die die beobachtete Beschleunigung, Richtungsaenderung.

Wurdest du den Vorgang vom Elektronaus betrachten wuerde sich eine zusaezliche Kraft ergeben. Aber warum sollte man es sich schwerer machen als noetig ?
Gruesse

JoAx
11.06.11, 14:09
Hallo,

meine Frage bezieht sich auf bewegte Ladungsträger im Magnetfeld. Ich weiss, dass
die Lorentzkraft auf die Teilchen als resultierende Kraft wirkt und sie
auf eine Kreisbahn ablenkt und so als Zentripetalkraft wirkt.
Nun muss es nach Newton eigentlich eine Gegenkraft zu der Lorentzkraft geben, welche
vom Kreismittelpunkt aus auf die Teilchen wirkt und vom Betrag gleich groß ist.

Meine Frage lautet; welche Kraft wirkt in diesem Fall als Gegenkraft?

Vielen Dank


George

Hallo, George!

Die gesuchte Kraft ist die Trägheitskraft. Der Umfang der (Kreis-) Bahn hängt u.a. auch von der Masse des Testteilchens ab.


Gruss, Johann

richy
11.06.11, 15:25
Die gesuchte Kraft ist die Trägheitskraft.
Warum Kraft ?
F= m*a
Links steht die Ursache und rechts die Wirkung auf eine traege Masse m. Diese aendert ihren Geschwindigkeitsvektor.
Die Geschwindigkeitsaenderung der Masse wird beobachtet und das ist keine Kraft, sondern das Resultat einer (Summe von) Kraeften.
Gaebe es eine Gegenkraft zur Lorentzkraft, so waere die Summe aller Kraefte gleich Null und die Ladung wuerde sich weiter unveraendert geradeaus bewegen. So wie wenn man auf der Ladung mitreisen wuerde. In dem Fall muss eine Gegenkraft existieren.

Trägheits- oder Scheinkräfte sind die Kräfte, die auf Körper nur wirken, wenn man sie nicht in einem Inertialsystem, sondern in einem beschleunigten Bezugssystem beschreibt.

George_05
12.06.11, 10:13
Hallo,

ersteinmal vielen Dank für die Diskussion,
bei meinen Überlegungen, schließe ich die Zentrifugalkraft aus, da sie eine Trägheitskrft bzw Scheinkraft ist. Zentripetalkraft und Zentrifugalkraft wirken auf den gleichen Körper und erfüllen nicht das 3 Newtonsche Axiom, wonach eine Kraft, die von einem Körper auf einen anderen ausgeübt wird, durch Wechselwirkung eine Gegenkraft entstehen lässt, die vom Betrag her gleich groß ist und in die entgegengesetzte Richtung wirkt., z. B. Gravitationskraft, ein Apfel fällt zu Boden, da die Erde eine Kraft auf den Apfel ausübt und diese eine gleichgroße auf den Apfel. Der Satellit übt eine Kraft auf die Erde aus und diese eine Kraft auf den Satelliten. Das ist unabhängig davon, ob es sich um ein Inertialsystem handelt oder um ein beschleunigtes System handelt.

Somit überlege ich, dass das Elektron eine Kraft auf den Magneten bzw. auf das Feld ausübt und dieser bzw. dieses auf das Elektron. Leider harke bei meinen Überlegungen daran, dass die Lorentzkraft bereits die resultierende Kraft (entstanden aus der Überlagerung aller Kräfte, die auf das Teilchen wirken) ist und somit müsste es eine weitere Kraft geben (vermutlich vom Feld auf das Elektron), damit das Elektron auf der Kreisbahn bleibt.

Gruß

George

Da steh ich nun ich armer Tor und bin so schlau als wie zuvor.

JoAx
12.06.11, 10:26
Hallo George!

Offensichtlich habe ich dich da missverstanden, "mit meiner Antwort". :)
Hoffentlich klappt das jetzt besser.


Da steh ich nun ich armer Tor und bin so schlau als wie zuvor.

Um die gesuchte Kraft zu finden, muss man wohl den Magneten im el. Feld der Ladung bewegt betrachten. Denke ich.


Gruss, Johann

George_05
12.06.11, 11:38
Hallo Johan,

ja, ich denke, dass ist der richtige Ansatz. Ich grübel noch mal ein wenig weiter.

Vielen Dank

Gruß

George

richy
12.06.11, 12:24
Hi Georg
Unter dem Freischneiden (oder Freimachen) von Körpern versteht man einen Vorgang in der Statik zur Berechnung der an ihnen angreifenden Kräfte und Momente.[1]
Das hattest du wohl gemeint.
http://de.wikipedia.org/wiki/Freischneiden
Um die gesuchte Kraft zu finden, muss man wohl den Magneten im el. Feld der Ladung bewegt betrachten.
Das denke ich auch und wird sicherlich keine einfache Aufgaben. Letztendlich wird die Kraft wohl auf die Erde uebertragen, die dadurch eine kleine Impulsaenderung erfaehrt.

JoAx
13.06.11, 10:32
Hallo richy!


Das denke ich auch und wird sicherlich keine einfache Aufgaben. Letztendlich wird die Kraft wohl auf die Erde uebertragen, die dadurch eine kleine Impulsaenderung erfaehrt.

Grundsätzlich hast du natürlich Recht, aber ich denke, dass es Goerge hier darum geht, diese Kraft überhaupt anzuschreiben. Für die Gravitation ist es einfach:

FG(r) = - G*M*m/r^2

m*a1 = - G*M*m/r^2 = M*a2

Lässt sich nun etwas ähnliches bei der Lorentz-Kraft

http://upload.wikimedia.org/math/0/0/5/00589749dd83593177cd3998495e62b3.png

aufschreiben?

Die Aufgabe dürfte schon seit "ein paar" Jährchen gelöst worden sein. :)

@George:
Willst du diese selbst lösen, oder weisst du nicht, wonach du googeln solltest?


Gruss, Johann

Benjamin
13.06.11, 22:02
meine Frage bezieht sich auf bewegte Ladungsträger im Magnetfeld. Ich weiss, dass
die Lorentzkraft auf die Teilchen als resultierende Kraft wirkt und sie
auf eine Kreisbahn ablenkt und so als Zentripetalkraft wirkt.


Zunächst handelt es sich bei der Ablenkung von geladenen Teilchen in einem Magnetfeld nicht zwangsläufig um eine Kreisbahn. Eine Kreisbahn entsteht z.B., wenn Geschwindigkeit der Ladung und Magnetfeld konstant sind und die B-Feld-Linien und der Geschwindigkeitsvektor normal aufeinander stehen.

Richtig ist aber, dass es sich bei der Lorentzkraft immer um eine Zentripetalkraft handelt.

Nun muss es nach Newton eigentlich eine Gegenkraft zu der Lorentzkraft geben, welche
vom Kreismittelpunkt aus auf die Teilchen wirkt und vom Betrag gleich groß ist.

Meine Frage lautet; welche Kraft wirkt in diesem Fall als Gegenkraft?

In der Tat gibt es diese Kraft. Sie wird in jedem Fall eine elektromagnetische Kraft sein, d.h. über die Gleichung F = q(E + vxB) beschrieben und ist somit entweder auf ein Magnetfeld, ein elektrisches Feld oder eine Mischung aus beidem zurückzuführen. By the way: Eigentlich ist der obige Ausdruck der vollständige für die Lorentzkraft. Es gibt also auch einen elektrischen und nicht nur magnetischen Anteil von ihr. Folglich ist die Gegenkraft zur Lorentzkraft selbst eine Lorentzkraft, und zwar immer.
Woher diese aber stammt und ob sie wirklich vom Kreismittelpunkt her rührt, ist eine andere Frage. Fakt ist, dass die Summe aller Kräfte genau der Lorentzkraft entgegen wirken, und somit vom Kreismittelpunkt weg zeigen.

Um deine Frage bis ins letzte Detail zu beantworten, müssen wir vorher klären, wie das Magnetfeld, das die Lorentzkraft verursacht, zustande kommt.

Dazu ein Beispiel: Sagen wir, wir schießen an einem stromdurchflossenen Leiter einen Elektronenstrahl vorbei, und zwar parallel zu diesem Leiter und parallel zur Stromrichtung. Der Elektronenstrahl wird durch die Lorentzkraft vom Leiter weg gekrümmt. Die entstehende Gegenkraft ist wie gesagt ebenso eine Lorentzkraft, da der Elektronenstrahl selbst einen Strom darstellt (bewegte Ladung ist). Diese Gegenkraft wirkt nun auf den Leiter und stoßt in ab. Elektronenstrahl und Leiter stoßen sich somit ab. Aufgrund dessen, dass der Leiter ab wesentlich mehr Masse besitzt, wird man die Lorentzkraft am Leiter sehr viel schwerer beobachten können als am Elektronenstrahl.

Benjamin
13.06.11, 22:23
Aja: Im Falle von Permanentmagneten, wo das Magnetfeld durch den Elektronenspin verursacht wird, wirkt die Gegenkraft auf diese Elektronen. Das heißt, ein geladenes Teilchen, das im Magnetfeld eines Permanentmagneten abgelenkt wird, verursacht auch eine Kraft, die auf den Magneten wirkt.

Für Magnetfelder, die durch stromführende Spulen (usw.) erzeugt werden, gilt dasselbe, was im Grunde schon aus meinem Beispiel im obigen Beitrag folgt. Ein bewegtes, geladenes Teilchen stellt immer einen Strom dar! Daraus folgt, dass es selbst ein Magnetfeld mit sich führt, welches mit anderen Magnetfeldern und Strömen wechselwirkt, also auch mit den Strömen in einer stromführenden Spule. ;)

Maxi
15.09.11, 23:19
Hallo,
natürlich hast Du vollkommen recht: Die Lorentzkraft, die auf das geladene Teilchen einwirkt, das sich senkrecht zu den Magnetfeldlinien bewegt, ist eine echte Kraft. Sie unterliegt damit in einem Inertialsystem auch dem dritten Newtonschen Gesetz „actio gegengleich reactio". Das Magnetfeld wird z.B. von einem stromdurchflossenen Helmholtzspulenpaar erzeugt und ist deshalb stark mit dem Spulenpaar verbunden. Des weiteren wirkt dieses Magnetfeld als Kraftübermittler für die Lorentzkraft auf das sich bewegende, geladene Teilchen (also auf den "Körper 2") ein, das dadurch (wie Du richtig sagst) auf eine Kreisbahn gezwungen wird.
Nun ist die alles entscheidende Frage: von welchem "Körper1" geht diese Kraft (die Lorenzkraft) aus?
Die naheliegenste Antwort darauf ist: "Körper 1" wird primär durch das Helmholtzspulenpaar realisiert. Dabei wird das Teilchen nicht zum Kreismittelpunkt gezogen (wie etwa ein Stein, der an eine Schnur gebunden im Kreis herumgeschleudert wird); das Spulenpaar schiebt das geladene teilchen vielmehr mittels Magnetfeld in Form der Lorentzkraft nach innen; Die Lorentzkraft übernimmt dabei die Funktion der Zentripedalkraft. Genau über denselben Kraftübermittler, über das Magnetfeld, wirkt nun der "Körper 2" (das geladene Teilchen) auf den "Körper 1" (das Spulenpaar) als "reactio" zurück. Man sollte nicht sagen (wie Du andeutest), dass die Gegenkraft vom Mittelpunkt des Kreises ausgeht; die Reaktionskraft geht schlicht vom geladenen Teilchen aus und wirkt seinerseits mittels des Magnetfeldes auf das Spulenpaar ein. Dabei ist die Gegenkraft in jedem Moment betragsgleich mit der Lorentzkraft und dieser entgegengesetzt. Hätte das Spulenpaar eine annähernd gleich kleine Masse wie das geladene Teilchen, würde das Spulenpaar aufgrund dieser "Gegenkraft" um das im Kreis laufende geladene Teilchen "herumeiern". Da die Massenverhältnisse aber nun mal so extrem unterschiedliche sind, wie sie sind, merkt man dem Spulenpaar diese Bewegung nicht an, da dieses außerdem noch am Tisch "verankert" ist. Aber "actio gegengleich reactio" ist physikalisch real vorhanden und erfüllt. Um es klar zu sagen: diese Reaktionskraft (oder Gegenkraft) hat absolut nichts mit einer Träheitskraft zu tun; genau so wenig wie mit der Zentrifugalkraft, die lediglich im „beschleunigten, rotierenden Bezugsystem“ von Nutzen ist und das dritte Newtongesetz nicht erfüllt.
Gruß, Maxi

Maxi
18.09.11, 10:39
Zunächst handelt es sich bei der Ablenkung von geladenen Teilchen in einem Magnetfeld nicht zwangsläufig um eine Kreisbahn. Eine Kreisbahn entsteht z.B., wenn Geschwindigkeit der Ladung und Magnetfeld konstant sind und die B-Feld-Linien und der Geschwindigkeitsvektor normal aufeinander stehen.

Richtig ist aber, dass es sich bei der Lorentzkraft immer um eine Zentripetalkraft handelt.

In der Tat gibt es diese Kraft. Sie wird in jedem Fall eine elektromagnetische Kraft sein, d.h. über die Gleichung F = q(E + vxB) beschrieben und ist somit entweder auf ein Magnetfeld, ein elektrisches Feld oder eine Mischung aus beidem zurückzuführen. By the way: Eigentlich ist der obige Ausdruck der vollständige für die Lorentzkraft. Es gibt also auch einen elektrischen und nicht nur magnetischen Anteil von ihr. Folglich ist die Gegenkraft zur Lorentzkraft selbst eine Lorentzkraft, und zwar immer.
Woher diese aber stammt und ob sie wirklich vom Kreismittelpunkt her rührt, ist eine andere Frage. Fakt ist, dass die Summe aller Kräfte genau der Lorentzkraft entgegen wirken, und somit vom Kreismittelpunkt weg zeigen.

Um deine Frage bis ins letzte Detail zu beantworten, müssen wir vorher klären, wie das Magnetfeld, das die Lorentzkraft verursacht, zustande kommt.

Dazu ein Beispiel: Sagen wir, wir schießen an einem stromdurchflossenen Leiter einen Elektronenstrahl vorbei, und zwar parallel zu diesem Leiter und parallel zur Stromrichtung. Der Elektronenstrahl wird durch die Lorentzkraft vom Leiter weg gekrümmt. Die entstehende Gegenkraft ist wie gesagt ebenso eine Lorentzkraft, da der Elektronenstrahl selbst einen Strom darstellt (bewegte Ladung ist). Diese Gegenkraft wirkt nun auf den Leiter und stoßt in ab. Elektronenstrahl und Leiter stoßen sich somit ab. Aufgrund dessen, dass der Leiter ab wesentlich mehr Masse besitzt, wird man die Lorentzkraft am Leiter sehr viel schwerer beobachten können als am Elektronenstrahl.

Ich finde, diese hier von Benjamin bereits am 14.07.2010 gegeben Erklärung ist die vernünftige und damit wohl richtige Lösung des Problems.
Damit ist meine Aussage in meinem letzten Beitrag eindeutig falsch: Genau über denselben Kraftübermittler, über das Magnetfeld, wirkt nun der "Körper 2" (das geladene Teilchen) auf den "Körper 1" (das Spulenpaar) als "reactio" zurück.
Nach Benjamin folgt vielmehr: Der sich bewegende "Körper 2" (das geladene Teilchen) erzeugt u.a. am Ort der Helmboltsspulen ein eigenes Magnetfeld, in dem die fließenden Ladungen der Spulen ihrerseits einer "Lorentzkraft 2" ausgesetzt sind. Dadurch werden diese Leitungselektronen von innen an die Leitungswände gedrückt. Und da sie diese Barriere nicht überwinden können, reißen sie die ganze Spulen mit sich. Und wären die Massen der beiden aktiven Körper nicht so verschieden groß, müsste man die Bewegung beider Körper im Prinzip registrieren können.
Auf völlig analoge Art erklärt man ja auch die "gegenseitige Abstoßung" bzw. "gegenseitige Anziehung" zweier nebeneinander liegender Leiter, in denen zwei Parallel- bzw. Gegenströmen fließen.

Gruß Maxi,
danke, hab wieder was dazu gelernt.

Maxi
18.09.11, 10:39
Zunächst handelt es sich bei der Ablenkung von geladenen Teilchen in einem Magnetfeld nicht zwangsläufig um eine Kreisbahn. Eine Kreisbahn entsteht z.B., wenn Geschwindigkeit der Ladung und Magnetfeld konstant sind und die B-Feld-Linien und der Geschwindigkeitsvektor normal aufeinander stehen.

Richtig ist aber, dass es sich bei der Lorentzkraft immer um eine Zentripetalkraft handelt.

In der Tat gibt es diese Kraft. Sie wird in jedem Fall eine elektromagnetische Kraft sein, d.h. über die Gleichung F = q(E + vxB) beschrieben und ist somit entweder auf ein Magnetfeld, ein elektrisches Feld oder eine Mischung aus beidem zurückzuführen. By the way: Eigentlich ist der obige Ausdruck der vollständige für die Lorentzkraft. Es gibt also auch einen elektrischen und nicht nur magnetischen Anteil von ihr. Folglich ist die Gegenkraft zur Lorentzkraft selbst eine Lorentzkraft, und zwar immer.
Woher diese aber stammt und ob sie wirklich vom Kreismittelpunkt her rührt, ist eine andere Frage. Fakt ist, dass die Summe aller Kräfte genau der Lorentzkraft entgegen wirken, und somit vom Kreismittelpunkt weg zeigen.

Um deine Frage bis ins letzte Detail zu beantworten, müssen wir vorher klären, wie das Magnetfeld, das die Lorentzkraft verursacht, zustande kommt.

Dazu ein Beispiel: Sagen wir, wir schießen an einem stromdurchflossenen Leiter einen Elektronenstrahl vorbei, und zwar parallel zu diesem Leiter und parallel zur Stromrichtung. Der Elektronenstrahl wird durch die Lorentzkraft vom Leiter weg gekrümmt. Die entstehende Gegenkraft ist wie gesagt ebenso eine Lorentzkraft, da der Elektronenstrahl selbst einen Strom darstellt (bewegte Ladung ist). Diese Gegenkraft wirkt nun auf den Leiter und stoßt in ab. Elektronenstrahl und Leiter stoßen sich somit ab. Aufgrund dessen, dass der Leiter ab wesentlich mehr Masse besitzt, wird man die Lorentzkraft am Leiter sehr viel schwerer beobachten können als am Elektronenstrahl.

Ich finde, diese hier von Benjamin bereits am 14.07.2010 gegeben Erklärung ist die vernünftige und damit wohl richtige Lösung des Problems.
Damit ist meine Aussage in meinem letzten Beitrag eindeutig falsch: Genau über denselben Kraftübermittler, über das Magnetfeld, wirkt nun der "Körper 2" (das geladene Teilchen) auf den "Körper 1" (das Spulenpaar) als "reactio" zurück.
Nach Benjamin folgt vielmehr: Der sich bewegende "Körper 2" (das geladene Teilchen) erzeugt u.a. am Ort von "Körper 1" (der Helmboltsspulen) ein eigenes Magnetfeld, in dem die fließenden Ladungen der Spulen ihrerseits einer "Lorentzkraft 2" ausgesetzt sind. Diese Kraft ist unsere gesuchte "Gegenkraft" (reactio). Dadurch werden diese Leitungselektronen von innen an die Leitungswände gedrückt. Und da sie diese Barriere nicht überwinden können, reißen sie die ganze Spulen mit sich. Und wären die Massen der beiden aktiven Körper nicht so verschieden groß, müsste man die Bewegung beider Körper im Prinzip registrieren können.
Auf völlig analoge Art erklärt man ja auch die "gegenseitige Abstoßung" bzw. "gegenseitige Anziehung" zweier nebeneinander liegender Leiter, in denen zwei Parallel- bzw. Gegenströmen fließen.

Gruß Maxi,
danke, hab wieder was dazu gelernt.

EMI
18.09.11, 11:48
Auf völlig analoge Art erklärt man ja auch die "gegenseitige Abstoßung" bzw. "gegenseitige Anziehung" zweier nebeneinander liegender Leiter, in denen zwei Parallel- bzw. Gegenströmen fließen.Richtig erklärt das die SRT von EINSTEIN Maxi,

schau hier mal rein:

http://www.quanten.de/forum/showpost.php5?p=52900&postcount=2
http://www.quanten.de/forum/showpost.php5?p=52973&postcount=33

Gruß EMI

Benjamin
18.09.11, 23:41
Richtig erklärt das die SRT von EINSTEIN Maxi,

schau hier mal rein:

http://www.quanten.de/forum/showpost.php5?p=52900&postcount=2
http://www.quanten.de/forum/showpost.php5?p=52973&postcount=33

Gruß EMI

Stimmt, EMI. Nur Vorsicht! Es kann unter Umständen zu Verwirrung führen, die Abstoßung zweier stromführender Drähte allein auf ein elektrisches Feld zurück zu führen, wenngleich es einwandfrei möglich ist.
Schwieriger wird das Ganze im Falle für beschleunigte Ladungen. Ich denke zwar, dass es möglich sein muss, ihre Wechselwirkung allein über ein E-Feld zu erklären, ich kenne dazu aber keine Abhandlung und weiß auch selbst nicht genau, wie das funktionieren soll.

EMI
19.09.11, 03:52
Schwieriger wird das Ganze im Falle für beschleunigte Ladungen.Stimmt Benjamin,

um Beschleunigungen einzubeziehen erweiterte EINSTEIN extra seine SRT zur ART.;)

1. Eine el.Ladung hat ein konstantes el.Feld welches auf andere el.Ladungen wirkt.

2. Eine gleichförmig bewegte el.Ladung erzeugt ein konstantes mag.Feld welches sich mit c ausbreitet und auf andere el.Ladungen wirkt die sich nicht mit der erzeugenden el.Ladung mitbewegen.

3. Eine beschleunigte el.Ladung erzeugt ein sich veränderndes mag.Feld welches sich mit c ausbreitet und seinerseits ein sich veränderndes el.Feld erzeugt welches sich mit c ausbreitet welches seinerseits ein sich veränderndes mag.Feld erzeugt was sich mit c ausbreitet usw. usw. und die auf andere el.Ladungen wirken die sich nicht mit der erzeugenden el.Ladung mitbeschleunigen. (el.mag.Welle)

Gruß EMI

Maxi
22.09.11, 09:43
Hallo Emi,

Richtig erklärt das die SRT von EINSTEIN Maxi,

schau hier mal rein:

http://www.quanten.de/forum/showpost.php5?p=52900&postcount=2
http://www.quanten.de/forum/showpost.php5?p=52973&postcount=33

A) habe zunächst erst einmal deinen ertsen Artikel gelesen. Eine faszinierende Sache, alles auf die Coulomb-Kraft zurückzuführen. Auf die Idee muss man aber erst einmal kommen, die elektrische Feldstärke mittels der Anzahl N der Feldlinien auszudrücken: |E| = N/L1*L2. N = 1 steht dann ja zunächst erst einmal für einen fest vereinbarten Feldstärkenwert |Eo|; oder sehe ich das falsch? Mir ist diese Art der Beschreibung nämlich völlig fremd.
Der rein formalen, rechnerischen Abhandlung kann ich in allen Einzelheiten folgen (hab alles nachgerechnet), aber in der allerletzten Schlussfolgerung verliere ich den Faden.
Doch zunächst: Bevor die Entwicklung des „Bio-Savartschen Gesetzes“ beginnt, versetzen wir uns also wieder ins Laborsystem. Die Ladungen der Elektronen q1 und q2 sind weiterhin Punktladungen. Die Längen L1 und L2 haben nichts mehr mit den Längen L1 und L2 aus obiger Gleichung |E| = N/L1*L2 zu tun. Denn in obiger Gleichung bilden die Längen L1 und L2 ein Rechteck, das senkrecht von den Feldlinien durchdrungen wird, die von q1 ausgehen. Hier liegen die beiden Längen parallel zueinander und es gilt L1 = v1*t und L2 = v2*t, wobei t der Zeit im Laborsystem entspricht. Nun werden die beiden Ströme einander angeglichen, was zur Folge hat, dass L1 = L2 = L, q1 = q2 = q und v1 = v2 gilt.
O.B.d.A. sei nun B = B2, die magnetische Flussdichte am Ort der Ladung q2, wobei B2 von J1 (der fließenden Ladung q1) verursacht wird. Der Abstand der beiden Leiter sei weiterhin r. Dann folgt mit den üblichen Definitionen: |H| = |- J*L/4*pi*r^2|. Wie hängt aber nun L physikalisch mit r zusammen?
Das übliche Verfahren zur Entwicklung der magnetischen Feldstärke [durch Integration über dem Bio-Savartschen Gesetz im Spezialfall eines unendlich langen, geraden Stromleiters ergibt doch: |H| = | J/2*pi*r|
Kannst Du mir weiterhelfen?
B) Feynman sagt in seinen Vorlesungen (ISBN 3-468-33701-7, Kap. 1. Seite 9) sinngemäß: Der Faktor c^2 trete deshalb in den Maxwellgleichungen auf, weil der Magnetismus in Wirklichkeit ein relativistischer Effekt der Elektrizität sei. Diese Äußerung spricht, denke ich, ebenfalls dafür, dass das magnetische Feld mit der SRT in den Griff zu bekommen sein müsste.
Ist es prinzipiell nicht so, wenn in einem Inertialsystem bei einem Experiment ein elektrisches Feld und zugleich ein magnetisches Feld vorhanden sind, lassen sich stets zwei weitere Inertialsysteme berechnen (bzw. bestimmen), in denen für dieses Experiment entweder das elektrische Feld oder das magnetische Feld total verschwindet?

Gruß, Maxi

Benjamin
22.09.11, 10:18
Ist es prinzipiell nicht so, wenn in einem Inertialsystem bei einem Experiment ein elektrisches Feld und zugleich ein magnetisches Feld vorhanden sind, lassen sich stets zwei weitere Inertialsysteme berechnen (bzw. bestimmen), in denen für dieses Experiment entweder das elektrische Feld oder das magnetische Feld total verschwindet?


Es lässt sich immer ein Bezugssystem finden, in dem das B-Feld verschwindet. Und das ist immer das Ruhesystem der Ladung, die du zu beschreiben wünscht. Setzt du dich in die Ladung, bewegst dich mit ihr mit, dann ruht sie selbstverständlich zu dir. Magnetfelder haben aber keinen Einfluss auf ruhende Ladung weil die Kraft, die ein Magentfeld auf eine Ladung ausübt immer der Geschwindigkeit proportional ist, mit v x B. (eigentlich immer proportional zur Geschwindigkeit senkrecht zu den B-Feldlinien) Für eine ruhende Ladung existiert sozusagen gar kein Magnetfeld! D.h. aus Sicht der Ladung gibt es kein Magnetfeld, sie reagiert immer nur auf elektrische Felder.
Elektrische Felder kannst du nur in Spezialfällen wegtransformieren, und zwar immer dann, wenn es ein Bezugssystem gibt, in dem sich die Wirkung von positiver und negativer Ladung aufheben. Die Wirkung des E-Feldes eines Elektrons alleine lässt sich niemals wegtransformieren, egal, welches Bezugssystem du wählst!

EMI
22.09.11, 11:55
Die Längen L1 und L2 haben nichts mehr mit den Längen L1 und L2 aus obiger Gleichung |E| = N/L1*L2 zu tun.Richtig Maxi,

hmm, das ist der Schreibmöglichkeit hier geschuldet, sorry.
Das L, L1 und L2 (bei BIOT/SAVART) hätte ich anders schreiben müssen, es sind dort natürlich die Länge der Leiterstücke.

Das kleine l wäre da besser gewesen, ist aber dann wieder mit dem großen I für den el.Strom leicht zu verwechseln.
Ich hätte s für die el.Leiterlänge nehmen sollen, sorry.

Gruß EMI

PS: Den Winkel sin α zwischen Stromflußrichtung und Abstandsrichtung (r) der Leiter hatte ich auch unterschlagen (gleich 1 gesetzt), da α=90° bei parallelen Leitern ist.

EMI
22.09.11, 11:58
Elektrische Felder kannst du nur in Spezialfällen wegtransformieren, und zwar immer dann, wenn es ein Bezugssystem gibt, in dem sich die Wirkung von positiver und negativer Ladung aufhebenOder wenn die Quelle des el.Feldes nicht eine el.Ladung sondern ein sich änderndes mag.Feld ist Benjamin.

Gruß EMI

Benjamin
22.09.11, 13:25
Oder wenn die Quelle des el.Feldes nicht eine el.Ladung sondern ein sich änderndes mag.Feld ist Benjamin.


Die Ursache eines mag. Feldes ist immer eine bewegte Ladung. Ein sich änderndes Magnetfeld ist insofern auch von einer elektr. Ladung verursacht. Es gibt kein Magnetfeld ohne elektr. Ladung. Eine umgekehrte Analogie gibt es nicht, weil es keine mag. Ladung gibt.

EMI
22.09.11, 15:00
Die Ursache eines mag. Feldes ist immer eine bewegte Ladung. Ein sich änderndes Magnetfeld ist insofern auch von einer elektr. Ladung verursacht. Es gibt kein Magnetfeld ohne elektr. Ladung. Eine umgekehrte Analogie gibt es nicht, weil es keine mag. Ladung gibt.Alles richtig Benjamin,

ich meinte aber folgendes:

Eine beschleunigte el.Ladung (Schwingkreis) erzeugt ein sich zeitlich änderndes mag.Feld. Ursache ist hier die beschleunigte el.Ladung, klar.

Dieses, von der beschleunigten el. Ladung erzeugte, sich zeitlich änderndes mag.Feld ist nun seinerzeits Ursache und Quelle eines weiteren sich zeitlich ändernden el.Feldes.
Dieses sich zeitlich änderndes el.Feld hat als direkte Ursache keine el.Ladung sondern das vorher erzeugte sich ändernde mag.Feld ist dessen unmittelbare Ursache.

Dieses zweite sich zeitlich ändernde el.Feld ist nun wiederum die Ursache für die Erzeugung eines weiteren sich zeitlich ändernden mag.Feldes.
Usw. usw. usw., bis die ursächlichste Quelle für das Alles, die el.Ladung aufhört zu beschleunigen.

Das Ganze nennt man elektromagnetische Welle.

Gruß EMI

Maxi
22.09.11, 18:44
Es lässt sich immer ein Bezugssystem finden, in dem das B-Feld verschwindet. Und das ist immer das Ruhesystem der Ladung, die du zu beschreiben wünscht. ( .... )
Elektrische Felder kannst du nur in Spezialfällen wegtransformieren, und zwar immer dann, wenn es ein Bezugssystem gibt, in dem sich die Wirkung von positiver und negativer Ladung aufheben. Die Wirkung des E-Feldes eines Elektrons alleine lässt sich niemals wegtransformieren, egal, welches Bezugssystem du wählst!

Danke Benjamin, das scheint mir plausibel zu sein.

Die Zusatzinformation von Emi an Dich übersteigt leider meinen Horizont.

Gruß, Maxi

Maxi
22.09.11, 20:34
(....)
Gruß EMI

PS: Den Winkel sin α zwischen Stromflußrichtung und Abstandsrichtung (r) der Leiter hatte ich auch unterschlagen (gleich 1 gesetzt), da α=90° bei parallelen Leitern ist.

Danke EMI,

aufgrund dieser Deiner abschließenden Anmerkung hab ich`s endlich kapiert.

Ich hatte mir bislang eingebildet, Dein Ergebnis |H| = |- J*L/4*pi*r^2| entspräche dem allgemeinen Ausdruck des Integrationsergebnisses |H| = | J/2*pi*r|.
In Wirklichkeit ist es das reinrassige -- physikalisch absolut korrekte -- Bio-Savartsche Gesetz, wie Du es in Deinem Artikel angekündigt hattest, das lediglich an die spezielle, vorgegebene Situation angepasst ist. Nach entsprechender Änderung Variablenbezeichnungen, etwa so:

|dH| = |-J*|r|*sin 90°*d|s|/4*pi*|r|^3

Und da wir nur ein einziges Ladungsteilchen q1 haben, ist |dH| mit |H| und d|s| lässt sich durch die gesamte Leiterlänge
|s| erstzen. Somit entsteht das kurzgefasste Bio-Savartsche Gesetz in der Form
|H| = |-J*|s|/4*pi*|r|^2.

Alles klar und nochmals Danke.

Gruss, Maxi

PS: Aufgrund der beiden Näherungen, die in Deiner Abhandlung vorkommen, tut sich mir eine neue Frage auf:

Das magnetische Feld H bzw. die magn. Flussdichte B werden doch durch die Maxwell-Gleichungen (einschließlich dem Bio-Savartschen Gesetz) exakt beschrieben, so exakt, dass sie auch durch die SRT in ihrer Genauigkeit nicht verbesserungsbedürftig sind. Weshalb gelangt man dann aber zu einem korrekten und sauberen Bio-Savartschen Gesetz, obwohl man doch auf dem Weg von (bzw. mit) der sauberen SRT beginnend, bis zur entgültigen Formel ettliche Rundungsfehler bei der Reihenentwicklung (durch Weglassung höherer Potenzen von (v/c)) in Kauf nimmt? Die klassische kinetische Energie m*v^2/2 ist ja auch nur deshalb nicht die vollständige kinetische Energie, weil sie nur das erste Glied der Reihenentwicklung darstellt.
Mit anderen Worten:
Ist die Größe "magnetisches Feld" nicht nur bloss ein relativistisches Korrekturglied, wie Feynman betont; sondern sogar ein mangelhaftes, da nur ein annähernd genaues?

EMI
22.09.11, 22:28
Ist die Größe "magnetisches Feld" nicht nur bloss ein relativistisches Korrekturglied, wie Feynman betont; sondern sogar ein mangelhaftes, da nur ein annähernd genaues?Du kannst einem ganz schön zusetzen Maxi, Du meine Güte.:eek:

Ich denke nicht, das die Näherungen dazu führen, dass das mag.Feld nur angenähert genau ist.
Ohne es ohne Näherungen durchgerechnet zu haben gehe ich davon aus, dass der Unterschied im Ergebnis erst weit hinterm Komma sichtbar wird.

Vieleicht äußert sich mal ein Matheexperte (Hawkwind:)) dazu, ich bin in Mathe recht wenig bewandert, sorry.

Näherungen (vor allem in der ART) führe ich immer dann ein, wenn ich erkenne, das ich ohne diese nicht mehr weiterkomme.
Quasi den Weg des geringsten Widerstandes suchen. Für mich natürlich.:cool:

Mir kam es auf etwas anderes an, als ich die Rechnung einst hier einstellte.

1. Wenige wissen/wussten, dass das mag.Feld nichts weiter wie eine Zusatzkomponente (ich sage gern Scheinfeld dazu;)) des el.Feldes ist.
Diese ganz klare physikalische Tatsache erkennt man erst mit der SRT.

2. Es ist ein weit verbreiteter Irrtum, das sich relativistische Effekte erst bei Geschwindigkeiten nahe c bemerkbar machen.
Die Geschwindigkeit der Elektronen im Leiter ist eher nahe 0 als nahe c:

http://www.quanten.de/forum/showpost.php5?p=53411&postcount=157

Der relativistische Effekt entsteht hier wegen der riesigen Zahl der Elektronen und das summiert sich halt auf und wir können mit bloßem Auge sehen:
Die Drähte ziehen sich ja an oder stossen sich ja ab.

Gruß EMI

Maxi
26.09.11, 12:23
Zum Thema "parallel verlaufende Ströme" ziehen sich an bzw. stoßen sich gegenseitig ab:

Richtig erklärt das die SRT von EINSTEIN Maxi,

schau hier mal rein:

http://www.quanten.de/forum/showpost.php5?p=52900&postcount=2

Gruß EMI

Hallo EMI,

soll man deine Entwicklung des BIO-SAVARTSCHEN GESETZES bezogen auf ein einzelnes Elektron einfach so brach liegen lassen? Das Fantastische an der ganzen Sache ist ja, dass mit dem Bio-Savartschen Gesetz automatisch auch zugleich das gesamte Magnetfeld mitgeliefert wird und somit jedwede Art von Aufsummierung unnötig ist.

|dH| = |H| = |-J*|r|*sin 90°*d|s|/4*pi*|r|^3 =|-e*v*sin(alpha)/4*pi*|r|^2

Ich geh' davon aus, es stimmt so!

Im Anhang habe ich dazu drei Skizzen angefertigt.
Auch wenn diese schwachen Felder in der Realität nicht direkt nachgewiesen werden können. Dein Gedanke ermöglicht jedoch - auch dies finde ich dabei toll - dass man nicht nur ein fremdes Magnetfeld auf ein bewegtes Elektron wirken lassen kann, sondern dass einfach zwei Magnetfelder, das fremde Feld und das des Elektrons, mit einander wechselwirken.

Gruß, Maxi

EMI
26.09.11, 20:03
...und somit jedwede Art von Aufsummierung unnötig ist.Ich weis nicht so recht was Du damit meinst Maxi,

die "Aufsummierung der Längenkontraktion" wird bei der Vielzahl der Elektronen und Atomkernen in den el.Leitern zum makroskopischen Effekt, trotz Geschwindigkeiten nahe 0.

Einzelnes Elektron, was soll sich da aufsummieren?

Gruß EMI

PS: Ich habe die nächsten Wochen kaum Zeit, sorry Maxi.
Kann ein Anderer übernehmen??
Maxi, Du bist gut und eine Bereicherung fürs Forum!

Nach PS: Ich lade mir keine PDF's usw. herunter, versuch deine Gedanken und Skizzen hier direkt einzustellen.

Benjamin
26.09.11, 22:24
Eine beschleunigte el.Ladung (Schwingkreis) erzeugt ein sich zeitlich änderndes mag.Feld.


Die Klammer irritiert mich hier. Jede beschleunigte Ladung strahlt in Form einer em Welle. Es muss sich um keinen Schwingkreis handeln und eine einzelne Ladung alleine kann gar kein Schwingkreis sein.


Dieses, von der beschleunigten el. Ladung erzeugte, sich zeitlich änderndes mag.Feld ist nun seinerzeits Ursache und Quelle eines weiteren sich zeitlich ändernden el.Feldes.
Dieses sich zeitlich änderndes el.Feld hat als direkte Ursache keine el.Ladung sondern das vorher erzeugte sich ändernde mag.Feld ist dessen unmittelbare Ursache.

Dieses zweite sich zeitlich ändernde el.Feld ist nun wiederum die Ursache für die Erzeugung eines weiteren sich zeitlich ändernden mag.Feldes.
Usw. usw. usw., bis die ursächlichste Quelle für das Alles, die el.Ladung aufhört zu beschleunigen.

Das Ganze nennt man elektromagnetische Welle.

Ja, ich kenne diese Erklärung. Ein sich änderndes E-Feld induziert ein B-Feld, dieses ändert sich ebenso und induziert wieder ein E-Feld, das sich wiederum ändert und wieder ein B-Feld inudziert, usw. Und das wäre dann eine elektromagnetische Welle.
Das ist eine Erklärung wie man sie gerne in Schulbüchern liest. Sie ist aber falsch, zumindest für das Fernfeld. Fakt ist, dass die mag. und elektr. Amplitude einer em Welle im Fernfeld völlig in Phase sind. Dh. E- und B-Feld erreichen gleichzeitig ihr Maximum und gleichzeitig ihr Minimum. Es ist nicht so, dass zuerst ein E-Feld da ist, aus dem dann ein B-Feld entsteht, um sich dann quasi wieder in ein E-Feld zu verwandeln. Ich schreibe dem B-Feld keine autonome Existenz vor. Es ist die relativistische Korrektur des E-Feldes.

richy
27.09.11, 00:19
Jede beschleunigte Ladung strahlt in Form einer em Welle.

Das wuerde ich nicht blind unterschreiben. Am Beispiel einer frei fallenden Ladung kann man sehen, dass entscheidend ist, dass die Ladung gegen ihr eigenes Feld Arbeit verrichten muss, damit eine EM Welle abgestrahlt wird.
Die Aenderung des Feldes ist das Entscheidende.
Wenn das Feld somit "mitfaellt" ist es fraglich, ob eine EM Welle abgestrahlt wird. Und das fuehrt zu der Fragestellung in welcher Form ein Feld mit der Ladung mitfallen koennte, ohne dass es sich veraendert. Das waere nur dann gegeben wenn elektrische Monopole, positive, negative Ladungen tatsaechlich unabhaengig voneinander existieren. Und das fuehrt zu der Frage ob folgendes idealiserte Feld ueberhaupt existiert :
http://upload.wikimedia.org/wikipedia/commons/thumb/1/19/LineasCampo.svg/220px-LineasCampo.svg.png
Die Feldlinien enden im Unendlichen. Befindet sich nirgends ein entgegengesetzter Ladungstraeger , dann wird diese dargestellt Ladung nicht Strahlen wenn sie sich beschleunigt bewegt. Das ist eine Erklärung wie man sie gerne in Schulbüchern liest. Sie ist aber falsch, zumindest für das Fernfeld.
Den Einwand fuer das Fernfeld kenne ich auch. Aber ich meine es liegt eher an der zu einfachen geometrischen Vorstellung die man hier zugrunde legt. Ein gewisser Herr Friebe hat mal ein entsprechendes Bildchen gemalt wie es eben nicht ist. So eine EM Welle aehnelt mehr einer Zwiebel. Aber von der Beschreibung passt das in etwa schon.
Die Herleitung der Wellen oder Telegraphenleitung wir uebrigends durch die Beziehung rot(rot(v))=grad(div(v))-Laplace(v) sehr einfach.
http://www-user.tu-chemnitz.de/~benner/Lehre/HM2/divrotgrad.pdf
Auch ohne jedliche Naeherung. Das PDE System wird dann zu einer PDE fuer eine Variable. Z.B. E und man erkennt sofort, dass c0 sich elementar aus den Maxwellgleichungen ergibt. Das ist lediglich ein Parameter der Wellengleichung. Und im Grunde ist es ein Wunder, dass c0 experimentell so genau in Uebereinstimmung mit den Maxwellgleichungen bestaetigt wird. Denn die Maxwellgleichungen sind linear. In der Natur gibt es aber in der Regel keine Linearitaet, denn irgenwann muss eine Saettigung eintreten. Vielleicht sind die Maxwellgleichungen hier aber tatsaechlich eine Ausnahme.

Fehlt hier nicht ein Integralzeichen ?
|dH| = |H|

die "Aufsummierung der Längenkontraktion" wird bei der Vielzahl der Elektronen und Atomkernen in den el.Leitern zum makroskopischen Effekt, trotz Geschwindigkeiten nahe 0.

Und damit waere es schon eine grosse Ignoranz dennoch zu meinen, dass die Effekte der RT lediglich Mathematik waeren. Die Raumzeit keine Entitaet.

Gruesse

EMI
27.09.11, 01:05
Und damit waere es schon eine grosse Ignoranz dennoch zu meinen, dass die Effekte der RT lediglich Mathematik waeren. Die Raumzeit keine Entitaet.Und genau auf dieser Basis kommt man sich (vielleicht) wieder näher. Vielleicht.

Gruß EMI

PS: Nun muss ich aber los, der Flieger wartet.

Maxi
28.09.11, 09:16
Ich weis nicht so recht was Du damit meinst Maxi,

die "Aufsummierung der Längenkontraktion" wird bei der Vielzahl der Elektronen und Atomkernen in den el.Leitern zum makroskopischen Effekt, trotz Geschwindigkeiten nahe 0.

Einzelnes Elektron, was soll sich da aufsummieren?

Wenn du nicht verstehst, was ich damit meine, dann kann dies im Prinzip eigentlich nur bedeuten, dass auch ich dich -- trotz meiner Euphorie -- nicht in allen Punkten verstanden habe. Ich lass' mir nochmal alles durch den Kopf gehen und melde mich dann bei Zeiten wieder; soll heißen: ich möchte nicht drängen.

Gruß, Maxi

Maxi
27.10.11, 09:42
Ich weis nicht so recht was Du damit meinst Maxi, ...

Einzelnes Elektron, was soll sich da aufsummieren?


Nach PS: Ich lade mir keine PDF's usw. herunter, versuch deine Gedanken und Skizzen hier direkt einzustellen.

Hallo Emi,

ich meine damit lediglich die Anwendung des Biot-Savartschen Gesetzes auf ein einzelnes bewegtes Elektron, das sich mit einem magnetischen Vektorfeld umgibt.
Wenn Du Lust hast, schau einfach in meinen Anhang.
Du möchtest Dir keine PDF`s herunterladen? Entschuldige, dies ist nicht notwendig: Du brauchst den Anhang nur zu öffnen. Verzeih`, die LaTex-Codesprache möchte ich mir nicht antun.
Nichts für ungut.
Gruss
Maxi